View Single Post
Old 08-04-2008, 01:34 PM   #8 (permalink)
Gregte
EcoModding Apprentice
 
Join Date: May 2008
Location: South Dakota
Posts: 117

GMC Sonoma - '94 GMC Sonoma
90 day: 36.97 mpg (US)
Thanks: 0
Thanked 4 Times in 4 Posts
I just figured out how to measure how much energy the pump is using, when NOT actually supplying energy to steer the wheels.

With a cold vehicle measure the temp of the pump. then start the engine and run it at say 2000 RPM for 5 minutes. Now again measure the temperature of the pump. It should be warmer. The amount warmer is the amount of energy it is wasting. Now you need to weigh the pump to get a total mass of metal, and separately weight the fluid that is in the pump (not in the rest of the system).

The metal of the pump can probably be reasonably correctly assumed to have a specific heat value of 0.12 (that of steel). This means it will take 0.12 BTUs to raise the temperature of 1 pound of it 1 degree F. If the pump weighed 10 pounds and if its temperature rose 5 degrees F in 5 minutes (if, if, if...), that would mean 1.2 BTUs were generated in 5 minutes. That is a rate of 72 BTU/hour. That equals 0.028297063 horsepower.

The above did not take into account the specific heat of the PS fluid in the pump but it would be similarly measurable and then just add it to the above. The fluid in the pump would be a rather small contributor to the whole considering the specific heat of oil is much lower than steel and the total weight of the fluid is much less I am sure.

So, about 3% of a HP if the above "ifs" are correct. In other words, to be 100% of a HP (1 HP) it would have to raise the temp 33 times as high, or do it 33 times as quickly or the pump would have to weigh 33 times as much OR any combination of the above.

To do this test correctly, any potential for heat gain and loss, other than that being generated in the pump, would have to be isolated. You would have to put insulation around the entire pump for the duration of the test with a temp probe affixed to the pump.

Please feel free to correct my math as I am doing this a bit hurriedly on my lunch break.
  Reply With Quote